Skip to Main Content

June 2007, Logical Reasoning 2, Question 17



Question 17. When a question asks you for an assumption required by the argument, it's a necessary assumption question. Necessary assumption questions ask us to find an assumption that the argument requires. Something that if it's not true, the argument can't work.

The evidence won't be able to prove the conclusion. In order to find the argument's assumptions, you need to understand the argument, which means breaking it down into its Inclusion and evidence. The conclusion of this argument is found in its first sentence. When exercising the muscles in your back, you need to make sure to exercise the muscles on both sides of the spine equally, if you want to maintain a healthy back.

The next two sentences give you evidence. The first one begins with the evidence keyword after all. After all balanced muscle development is needed to maintain a healthy back. And one more piece of evidence, with the evidence keyword, since. Since the muscles on opposite sides of the spine must pull equally in order to keep the back in proper alignment and protect the spine.

Now there is a subtle concept shift here between the evidence and the conclusion, which is where the assumption is going to be found. The evidence says that you need balanced muscle development to keep a healthy back. Because you need your muscles to be able to pull on both sides of your spine equally to have a healthy back. The conclusion isn't that you need balance muscle development.

But rather you need to exercise both sides of your back equally. So for this argument to work, we need to believe that exercising the muscles on both sides of your back equally, will tend to lead to balance muscle development. If that's not the case, the argument won't make sense. The evidence won't prove the conclusion. The fact that you need balance muscle development won't prove that you should do these equal side exercises.

So go to the answer choices looking for one of those assumptions, answer choice A. Now A is a mistaken version of one of the pieces of evidence we were given. We were told that you need the muscles working equally in order to keep the back in proper alignment. This answer choice is saying that having the muscles working evenly, will be enough to keep the back in proper alignment.

That's confusing necessity and sufficiency, saying that the back requires these muscles pulling evenly. It's not the same thing as saying that if it's pulling evenly the back is going to be okay. So the argument doesn't require believing a garbled version of one of its premises, so this is not our answer.

Go to B. Answer choice B is exactly what we're looking for. If working both sides unequally didn't tend to lead to unhealthy muscle development. Then working both sides equally wouldn't be required for the healthy back. And then the argument would fall apart.

So B is needed by the argument. It is our answer. We can glance at the other answers just to see why they're wrong. Answer choice C ends up being wrong for the same reason that answer choice A does, it's a necessity sufficiency problem. This argument is trying to prove that exercising your muscles equally is needed for a healthy back.

This answer choice says that if you do it, you will have a healthy back. That's confusing a requirement with a guarantee. So it's not our answer. Answer choice D is wrong because it goes way too far. The argument just trying to prove that you need this development in order to have a healthy back.

This says if you don't do it evenly, you're going to have a back that's irreparably damaged. That's way more than the argument requires. answer choice E. Well, no particular exercise regimen is required for this argument to work. Just that whatever exercises you do, and however frequently you do them, it involves exercising the muscles equally on both sides of the spine.

So it doesn't have to be daily for this argument to work doesn't have to be monthly, weekly every other day. There's no specific frequency it needs to be in order for the argument to work. So answer choice E is not our answer. The answer was B.

Read full transcript